Chief Complaint: A 28-year-old woman presents to the clinic with complaints of intermittent blurry vision and weakness in her right leg over the past few weeks. History of Present Illness: The patient reports that about a month ago, she experienced pain and decreased vision in her left eye, which gradually improved over two weeks. More recently, she has noticed a tingling sensation in her right leg, which progressed to mild weakness, making it difficult for her to walk long distances. She also reports episodes of fatigue and occasional difficulty controlling her bladder. She denies any fever, recent infections, or trauma. Past Medical History: No known chronic illnesses No prior neurological conditions No family history of similar symptoms Review of Systems: Neurologic: Numbness, weakness in the right leg, occasional imbalance Ophthalmologic: Blurry vision in the left eye, previously painful eye movements Genitourinary: Episodes of urinary urgency Physical Examination: Vital Signs: Normal Neurologic Exam: Decreased visual acuity in the left eye, relative afferent pupillary defect (RAPD) Hyperreflexia in the right leg Positive Lhermitte’s sign (electric shock-like sensation down the spine when flexing the neck) Mild difficulty with tandem gait Diagnostic Workup: MRI Brain and Spine: Multiple periventricular and spinal white matter lesions with contrast enhancement (suggestive of active demyelination). Lumbar Puncture (CSF Analysis): Presence of oligoclonal bands and increased IgG index. Visual Evoked Potentials (VEPs): Delayed response in the left optic nerve. What is the most likely diagnosis?

A 45-year-old individual diagnosed with Myasthenia Gravis (MG) undergoes a routine chest CT scan, which reveals the presence of a thymoma. The patient has been experiencing fluctuating muscle weakness, characteristic of MG, and has been managing the condition with acetylcholinesterase inhibitors and immunosuppressive therapy. Given the new finding of a thymoma, the healthcare team discusses potential treatment options, including surgical intervention. Question: Which of the following statements about thymectomy in this scenario is most accurate?